Difference between revisions of "1987 AJHSME Problems/Problem 9"

(New page: ==Problem== When finding the sum <math>\frac{1}{2}+\frac{1}{3}+\frac{1}{4}+\frac{1}{5}+\frac{1}{6}+\frac{1}{7}</math>, the least common denominator used is <math>\text{(A)}\ 120 \qquad \...)
 
m (Solution)
Line 7: Line 7:
 
==Solution==
 
==Solution==
  
We want <math>\text{LCM}(2,3,4,5,6,7)</math>, which is <math>840</math>, or choice <math>\boxed{\text{C}}</math>.  See [[Least common multiple|here]] if you don't know how to evaluate LCMs.
+
We want <math>\text{LCM}(2,3,4,5,6,7)</math>, which is <math>420</math>, or choice <math>\boxed{\text{C}}</math>.  See [[Least common multiple|here]] if you don't know how to evaluate LCMs.
  
 
==See Also==
 
==See Also==
  
 
[[1987 AJHSME Problems]]
 
[[1987 AJHSME Problems]]

Revision as of 22:39, 14 February 2009

Problem

When finding the sum $\frac{1}{2}+\frac{1}{3}+\frac{1}{4}+\frac{1}{5}+\frac{1}{6}+\frac{1}{7}$, the least common denominator used is

$\text{(A)}\ 120 \qquad \text{(B)}\ 210 \qquad \text{(C)}\ 420 \qquad \text{(D)}\ 840 \qquad \text{(E)}\ 5040$

Solution

We want $\text{LCM}(2,3,4,5,6,7)$, which is $420$, or choice $\boxed{\text{C}}$. See here if you don't know how to evaluate LCMs.

See Also

1987 AJHSME Problems